Home
Current Affairs January 2024

What is the correct answer?

4

Consider the following statements about Wholesale Price Index (WPI) :
1. It captures the price movement extensively and is, therefore, taken as an indicator of inflation.
2. The office of the Economic adviser (OEA) is entrusted with the function of compilation and publication of the weekly wholesale price index number in India.
Which of the statements given above is/are correct? [CDS 2009]

A. I only

B. II only

C. Both I and II

D. Neither I nor II

Correct Answer :

C. Both I and II


Related Questions

What is the correct answer?

4

Under the Constitution, the power to raise and disburse public funds:

A. has been vested in Union Government

B. has been divided between the Union and State Governments

C. has been given to the Finance Commission

D. has not been given to anyone

What is the correct answer?

4

To meet the growing needs for coins in the country, where does the Government propose to set up another mint?

A. Nasik

B. Hoshangabad

C. Dewas

D. Noida

What is the correct answer?

4

Consider the following statement:
The price of any currency in international market is determined by the:
1. WTO
2. Demand for goods/services provided by the country concerned
3. Inflation differential between the country concerned and its major trading partners
4. Stability of the government of the concerned country
Of these statements:

A. 1, 2, 3, and 4 are correct

B. 1, 2 and 4 are correct

C. 1, 3 and 4 are correct

D. 2, 3 and 4 are correct

What is the correct answer?

4

The period of high inflation and low economic growth is termed as: [RRB 1992]

A. stagnation

B. take-off stage in economy

C. stagflation

D. none of these

What is the correct answer?

4

M1 includes:

A. currency with public

B. demand deposit with bank

C. other deposits with RBI

D. all of the above

What is the correct answer?

4

Inflation is caused as a result of:

A. increase in money supply

B. fall in production

C. increase in money supply without a corresponding increase in production

D. decrease in money supply without a corresponding decrease in production

What is the correct answer?

4

Which of the following can be used for checking inflation temporarily?

A. Increase in wages

B. Decrease in money supply

C. Decrease in taxes

D. None of the above

What is the correct answer?

4

Which of the following is incorrect about convertibility?

A. The exchange rate should be determined by the forces of demand and supply of the currency

B. The exchange rate' would indicate the strength of the economy

C. It would discourage black market transactions

D. The RBI will be a direct player now rather than being an indirect one

What is the correct answer?

4

Of the various ways of financing government's investment expenditure, the least inflationary is : [PCS 1994]

A. foreign aid

B. deficit financing

C. taxation

D. public borrowing

What is the correct answer?

4

Inflation can be contained by:

A. surplus budget

B. increase in taxation

C. reduction in public expenditure

D. all the above

What is the correct answer?

4

The Indian Rupee is fully convertible:
1. In respect of Current Account of Balance of Payments
2. In respect of Capital Account of Balance of Payments
3. Into Gold
Which of these statements is/are correct ?

A. 1 only

B. 3 only

C. 1 and 2 only

D. 1, 2 and 3

What is the correct answer?

4

A very rapid growth in prices in which money loses its value to the point where even barter may be preferable is known as:

A. inflation

B. hyper-inflation

C. deflation

D. disinflation

What is the correct answer?

4

Which of the following accounts for Cost-Push Inflation?

A. Increase in money supply

B. Increase in indirect taxation

C. Increase in population

D. Increase in non-plan expenditure

What is the correct answer?

4

Devaluation of currency by a country is meant to lead to :
1. expansion of import trade
2. promotion of import substitution
3. expansion of export trade

A. 1 only

B. 2 and 3

C. 1 and 2

D. 1 and 4

What is the correct answer?

4

Which of the following governmental steps has proved relatively effective in controlling the double digit rate of inflation in the Indian economy during recent years?

A. Containing budgetory deficits and unproductive expenditure

B. Streamlined public distribution system

C. Enhanced rate of production of all consumer goods

D. Pursuing an export-oriented strategy

What is the correct answer?

4

Who among the following are not protected against inflation.? [Railways 1992]

A. Salaried class

B. Industrial workers

C. Pensioners

D. Agricultural farmers

What is the correct answer?

4

Inflation implies: [Railways 1994]

A. rise in budget deficit

B. rise in money supply

C. rise in general price index

D. rise in prices of consumer goods

What is the correct answer?

4

The best means of saving during inflation is to keep:

A. money

B. Government Bonds

C. equity

D. time deposits with Banks

What is the correct answer?

4

A steady increase in the general level of prices as a result of excessive increase in aggregate demand as compared to aggregate supply is termed as : [CDS 1999]

A. demand-pull inflation

B. cost-push inflation

C. stagflation

D. structural inflation

What is the correct answer?

4

Which of the following factors contributes to an inflationary trend?

A. 15% fall in production of industrial goods

B. 15% increase in prices of agricultural products

C. 15% increase in supply of money in the market

D. none of these

What is the correct answer?

4

Amongst the currency quotes USD/JPY, USD/Euro and USD/CAD, which one of the following is referred as the base currency for quotes? [Punjab & Sindh Bank 2011]

A. USD

B. JPY

C. Euro

D. None of these

What is the correct answer?

4

The Issue Department of the RBI maintains a _______ against printing of notes:

A. Minimum Reserve System

B. Proportional Reserve System

C. Proportional Gold Reserve System

D. Proportional Foreign Securities Reserve System

What is the correct answer?

4

Rupee was devalued by what percent in July 1991 ? [MP PCS 1990]

A. 18

B. 20

C. 22

D. 25

What is the correct answer?

4

Which of the following is wrongly matched?

A. Depression: Insufficient demand causing large scale unemployment of men and machinery over a long period of time

B. Recession: Reduction in demand and production/ investment over a short period of time

C. Stagflation: slow pace of economic activity due to falling prices

D. Boom: Rapid and all-round spurt in economic activity

What is the correct answer?

4

Which of the following Mahatma Gandhi series of currency notes issued by the RBI has a drawing of the 'Parliament House' depicted on it? [SSC Graduate 2003]

A. Rs. 500

B. Rs. 100

C. Rs. 50

D. Rs. 10

What is the correct answer?

4

Consider the following statements about Wholesale Price Index (WPI) :
1. It captures the price movement extensively and is, therefore, taken as an indicator of inflation.
2. The office of the Economic adviser (OEA) is entrusted with the function of compilation and publication of the weekly wholesale price index number in India.
Which of the statements given above is/are correct? [CDS 2009]

A. I only

B. II only

C. Both I and II

D. Neither I nor II

What is the correct answer?

4

The States' debt does not include:

A. loans from State Bank of India

B. loans from the Central Government

C. Provident Funds

D. treasury bills issued to international financial institutions

What is the correct answer?

4

Monetary policy is regulated by:

A. money lenders

B. Central Bank

C. private entrepreneurs

D. Government policy

What is the correct answer?

4

M3 includes:

A. M1 + T.D

B. M1 + post office saving deposit

C. M1 +- post office total deposit

D. M1 + national saving certificate

What is the correct answer?

4

Stagflation implies a case of: [CDS 1992]

A. galloping inflation

B. recession plus inflation

C. adverse balance of trade

D. rising wages and employment